Clarification on step in proof for Ratio test












1












$begingroup$


enter image description here





My thoughts: So apart from this step, the rest of the proof is fairly simple. Now in terms of lim sup, I know that it is just equal to the regular limit for convergent sequences and for bounded, divergent sequences it is what the "peaks" of the sequence converges towards. Of course there is a formal definition as well but I would like to have an intiutive understanding as well.



However I am not sure how to deduce the the inequality between the fraction and t from the definition of limsup... Can someone help me? Am I missing some key property of lim sup?










share|cite|improve this question









$endgroup$

















    1












    $begingroup$


    enter image description here





    My thoughts: So apart from this step, the rest of the proof is fairly simple. Now in terms of lim sup, I know that it is just equal to the regular limit for convergent sequences and for bounded, divergent sequences it is what the "peaks" of the sequence converges towards. Of course there is a formal definition as well but I would like to have an intiutive understanding as well.



    However I am not sure how to deduce the the inequality between the fraction and t from the definition of limsup... Can someone help me? Am I missing some key property of lim sup?










    share|cite|improve this question









    $endgroup$















      1












      1








      1





      $begingroup$


      enter image description here





      My thoughts: So apart from this step, the rest of the proof is fairly simple. Now in terms of lim sup, I know that it is just equal to the regular limit for convergent sequences and for bounded, divergent sequences it is what the "peaks" of the sequence converges towards. Of course there is a formal definition as well but I would like to have an intiutive understanding as well.



      However I am not sure how to deduce the the inequality between the fraction and t from the definition of limsup... Can someone help me? Am I missing some key property of lim sup?










      share|cite|improve this question









      $endgroup$




      enter image description here





      My thoughts: So apart from this step, the rest of the proof is fairly simple. Now in terms of lim sup, I know that it is just equal to the regular limit for convergent sequences and for bounded, divergent sequences it is what the "peaks" of the sequence converges towards. Of course there is a formal definition as well but I would like to have an intiutive understanding as well.



      However I am not sure how to deduce the the inequality between the fraction and t from the definition of limsup... Can someone help me? Am I missing some key property of lim sup?







      sequences-and-series convergence limsup-and-liminf






      share|cite|improve this question













      share|cite|improve this question











      share|cite|improve this question




      share|cite|improve this question










      asked Dec 20 '18 at 11:06









      CruZCruZ

      527




      527






















          2 Answers
          2






          active

          oldest

          votes


















          0












          $begingroup$

          Okay, If you consider simply the definition of lim sup The limit superior of ${x_{n}} $ is the smallest real number $b$ such that, for any positive real number $epsilon$ there exists a natural number $N$ such that



          $x_{n}<b+epsilon forall n>N$



          So, in your case if we simply consider in particular $epsilon=t-b$ there exists a natural number $N_epsilon$ such that $frac{|a_{k+1}|}{|a_{k}|} leq b+t-b=b forall k>N_epsilon$



          Even intuitively if you consider any number $t$ greater than the $limsup$ of your sequence. If there was no such natural number N such that $ forall kgeq N $ the sequence $frac{|a_{k+1}|}{|a_{k}|}$ was bounded above by $t$ this implies that the limsup is nessecarily greater than or equal to the number t as the sequence "peaks" beyond the value t for larger and larger values of k.






          share|cite|improve this answer









          $endgroup$





















            0












            $begingroup$

            Let $x_k = frac{|a_{k+1}|}{|a_k|}$.



            Assume that $limsup$ $x_k leq Q$ and let $t>Q$.



            By the definition of limsup, limsup $x_k =$ $inf_{n geq 1} sup_{m geq n}x_m$ $leq Q$. By the definition of the infimum this implies that there exists an N such that $sup_{m geq N}x_m leq t$ (note that $y_n = sup_{m geq n}x_m$ is a decreasing sequence). But this implies that $x_m leq t$ $forall m geq N$, as needed.






            share|cite|improve this answer









            $endgroup$













              Your Answer





              StackExchange.ifUsing("editor", function () {
              return StackExchange.using("mathjaxEditing", function () {
              StackExchange.MarkdownEditor.creationCallbacks.add(function (editor, postfix) {
              StackExchange.mathjaxEditing.prepareWmdForMathJax(editor, postfix, [["$", "$"], ["\\(","\\)"]]);
              });
              });
              }, "mathjax-editing");

              StackExchange.ready(function() {
              var channelOptions = {
              tags: "".split(" "),
              id: "69"
              };
              initTagRenderer("".split(" "), "".split(" "), channelOptions);

              StackExchange.using("externalEditor", function() {
              // Have to fire editor after snippets, if snippets enabled
              if (StackExchange.settings.snippets.snippetsEnabled) {
              StackExchange.using("snippets", function() {
              createEditor();
              });
              }
              else {
              createEditor();
              }
              });

              function createEditor() {
              StackExchange.prepareEditor({
              heartbeatType: 'answer',
              autoActivateHeartbeat: false,
              convertImagesToLinks: true,
              noModals: true,
              showLowRepImageUploadWarning: true,
              reputationToPostImages: 10,
              bindNavPrevention: true,
              postfix: "",
              imageUploader: {
              brandingHtml: "Powered by u003ca class="icon-imgur-white" href="https://imgur.com/"u003eu003c/au003e",
              contentPolicyHtml: "User contributions licensed under u003ca href="https://creativecommons.org/licenses/by-sa/3.0/"u003ecc by-sa 3.0 with attribution requiredu003c/au003e u003ca href="https://stackoverflow.com/legal/content-policy"u003e(content policy)u003c/au003e",
              allowUrls: true
              },
              noCode: true, onDemand: true,
              discardSelector: ".discard-answer"
              ,immediatelyShowMarkdownHelp:true
              });


              }
              });














              draft saved

              draft discarded


















              StackExchange.ready(
              function () {
              StackExchange.openid.initPostLogin('.new-post-login', 'https%3a%2f%2fmath.stackexchange.com%2fquestions%2f3047425%2fclarification-on-step-in-proof-for-ratio-test%23new-answer', 'question_page');
              }
              );

              Post as a guest















              Required, but never shown

























              2 Answers
              2






              active

              oldest

              votes








              2 Answers
              2






              active

              oldest

              votes









              active

              oldest

              votes






              active

              oldest

              votes









              0












              $begingroup$

              Okay, If you consider simply the definition of lim sup The limit superior of ${x_{n}} $ is the smallest real number $b$ such that, for any positive real number $epsilon$ there exists a natural number $N$ such that



              $x_{n}<b+epsilon forall n>N$



              So, in your case if we simply consider in particular $epsilon=t-b$ there exists a natural number $N_epsilon$ such that $frac{|a_{k+1}|}{|a_{k}|} leq b+t-b=b forall k>N_epsilon$



              Even intuitively if you consider any number $t$ greater than the $limsup$ of your sequence. If there was no such natural number N such that $ forall kgeq N $ the sequence $frac{|a_{k+1}|}{|a_{k}|}$ was bounded above by $t$ this implies that the limsup is nessecarily greater than or equal to the number t as the sequence "peaks" beyond the value t for larger and larger values of k.






              share|cite|improve this answer









              $endgroup$


















                0












                $begingroup$

                Okay, If you consider simply the definition of lim sup The limit superior of ${x_{n}} $ is the smallest real number $b$ such that, for any positive real number $epsilon$ there exists a natural number $N$ such that



                $x_{n}<b+epsilon forall n>N$



                So, in your case if we simply consider in particular $epsilon=t-b$ there exists a natural number $N_epsilon$ such that $frac{|a_{k+1}|}{|a_{k}|} leq b+t-b=b forall k>N_epsilon$



                Even intuitively if you consider any number $t$ greater than the $limsup$ of your sequence. If there was no such natural number N such that $ forall kgeq N $ the sequence $frac{|a_{k+1}|}{|a_{k}|}$ was bounded above by $t$ this implies that the limsup is nessecarily greater than or equal to the number t as the sequence "peaks" beyond the value t for larger and larger values of k.






                share|cite|improve this answer









                $endgroup$
















                  0












                  0








                  0





                  $begingroup$

                  Okay, If you consider simply the definition of lim sup The limit superior of ${x_{n}} $ is the smallest real number $b$ such that, for any positive real number $epsilon$ there exists a natural number $N$ such that



                  $x_{n}<b+epsilon forall n>N$



                  So, in your case if we simply consider in particular $epsilon=t-b$ there exists a natural number $N_epsilon$ such that $frac{|a_{k+1}|}{|a_{k}|} leq b+t-b=b forall k>N_epsilon$



                  Even intuitively if you consider any number $t$ greater than the $limsup$ of your sequence. If there was no such natural number N such that $ forall kgeq N $ the sequence $frac{|a_{k+1}|}{|a_{k}|}$ was bounded above by $t$ this implies that the limsup is nessecarily greater than or equal to the number t as the sequence "peaks" beyond the value t for larger and larger values of k.






                  share|cite|improve this answer









                  $endgroup$



                  Okay, If you consider simply the definition of lim sup The limit superior of ${x_{n}} $ is the smallest real number $b$ such that, for any positive real number $epsilon$ there exists a natural number $N$ such that



                  $x_{n}<b+epsilon forall n>N$



                  So, in your case if we simply consider in particular $epsilon=t-b$ there exists a natural number $N_epsilon$ such that $frac{|a_{k+1}|}{|a_{k}|} leq b+t-b=b forall k>N_epsilon$



                  Even intuitively if you consider any number $t$ greater than the $limsup$ of your sequence. If there was no such natural number N such that $ forall kgeq N $ the sequence $frac{|a_{k+1}|}{|a_{k}|}$ was bounded above by $t$ this implies that the limsup is nessecarily greater than or equal to the number t as the sequence "peaks" beyond the value t for larger and larger values of k.







                  share|cite|improve this answer












                  share|cite|improve this answer



                  share|cite|improve this answer










                  answered Dec 20 '18 at 12:33









                  Uday KhannaUday Khanna

                  21718




                  21718























                      0












                      $begingroup$

                      Let $x_k = frac{|a_{k+1}|}{|a_k|}$.



                      Assume that $limsup$ $x_k leq Q$ and let $t>Q$.



                      By the definition of limsup, limsup $x_k =$ $inf_{n geq 1} sup_{m geq n}x_m$ $leq Q$. By the definition of the infimum this implies that there exists an N such that $sup_{m geq N}x_m leq t$ (note that $y_n = sup_{m geq n}x_m$ is a decreasing sequence). But this implies that $x_m leq t$ $forall m geq N$, as needed.






                      share|cite|improve this answer









                      $endgroup$


















                        0












                        $begingroup$

                        Let $x_k = frac{|a_{k+1}|}{|a_k|}$.



                        Assume that $limsup$ $x_k leq Q$ and let $t>Q$.



                        By the definition of limsup, limsup $x_k =$ $inf_{n geq 1} sup_{m geq n}x_m$ $leq Q$. By the definition of the infimum this implies that there exists an N such that $sup_{m geq N}x_m leq t$ (note that $y_n = sup_{m geq n}x_m$ is a decreasing sequence). But this implies that $x_m leq t$ $forall m geq N$, as needed.






                        share|cite|improve this answer









                        $endgroup$
















                          0












                          0








                          0





                          $begingroup$

                          Let $x_k = frac{|a_{k+1}|}{|a_k|}$.



                          Assume that $limsup$ $x_k leq Q$ and let $t>Q$.



                          By the definition of limsup, limsup $x_k =$ $inf_{n geq 1} sup_{m geq n}x_m$ $leq Q$. By the definition of the infimum this implies that there exists an N such that $sup_{m geq N}x_m leq t$ (note that $y_n = sup_{m geq n}x_m$ is a decreasing sequence). But this implies that $x_m leq t$ $forall m geq N$, as needed.






                          share|cite|improve this answer









                          $endgroup$



                          Let $x_k = frac{|a_{k+1}|}{|a_k|}$.



                          Assume that $limsup$ $x_k leq Q$ and let $t>Q$.



                          By the definition of limsup, limsup $x_k =$ $inf_{n geq 1} sup_{m geq n}x_m$ $leq Q$. By the definition of the infimum this implies that there exists an N such that $sup_{m geq N}x_m leq t$ (note that $y_n = sup_{m geq n}x_m$ is a decreasing sequence). But this implies that $x_m leq t$ $forall m geq N$, as needed.







                          share|cite|improve this answer












                          share|cite|improve this answer



                          share|cite|improve this answer










                          answered Dec 20 '18 at 12:32









                          Sorin TircSorin Tirc

                          1,745213




                          1,745213






























                              draft saved

                              draft discarded




















































                              Thanks for contributing an answer to Mathematics Stack Exchange!


                              • Please be sure to answer the question. Provide details and share your research!

                              But avoid



                              • Asking for help, clarification, or responding to other answers.

                              • Making statements based on opinion; back them up with references or personal experience.


                              Use MathJax to format equations. MathJax reference.


                              To learn more, see our tips on writing great answers.




                              draft saved


                              draft discarded














                              StackExchange.ready(
                              function () {
                              StackExchange.openid.initPostLogin('.new-post-login', 'https%3a%2f%2fmath.stackexchange.com%2fquestions%2f3047425%2fclarification-on-step-in-proof-for-ratio-test%23new-answer', 'question_page');
                              }
                              );

                              Post as a guest















                              Required, but never shown





















































                              Required, but never shown














                              Required, but never shown












                              Required, but never shown







                              Required, but never shown

































                              Required, but never shown














                              Required, but never shown












                              Required, but never shown







                              Required, but never shown







                              Popular posts from this blog

                              Quarter-circle Tiles

                              build a pushdown automaton that recognizes the reverse language of a given pushdown automaton?

                              Mont Emei